Đến nội dung

tpdtthltvp

tpdtthltvp

Đăng ký: 26-01-2015
Offline Đăng nhập: 20-02-2024 - 20:31
***--

#651254 Cho x,y,z là các số thực dương CMR $(xy+yz+zx)[\frac{1}...

Gửi bởi tpdtthltvp trong 25-08-2016 - 20:32

Đây là BĐT $Iran \;\;\;\ 96.$ Bạn có thể xem ở link sau $(Pro.5):$   https://truongvoki9x...uongvoki_bn.pdf




#650949 Dựng điểm $M$ trong tam giác sao cho $MA+MB+MC$ đạt giá t...

Gửi bởi tpdtthltvp trong 23-08-2016 - 17:08

cho $\Delta$ABC nhọn, AB<AC. Hãy dựng điểm M nằm trong $\Delta$ sao cho:

MA+MB+MC đạt giá trị nhỏ nhất

MAMBMC.JPG

 Dựng các tam giác đều $AMN,ACP($như hình vẽ$).$ Dễ dàng chứng minh được: $\Delta AMC=\Delta ANP(c.g.c)$ suy ra $MC=NP.$

 Do đó: $MA+MB+MC=MB+MN+NP\geq BP$ không đổi. Dấu $"="$ xảy ra khi $B,M,N,P$ thằng hàng $\Leftrightarrow \widehat{AMB}=\widehat{AMC}=\widehat{BMP}=120^{\circ}$




#650913 Topic Ôn thi HSG 9 2015-2016 (Hình học)

Gửi bởi tpdtthltvp trong 23-08-2016 - 12:16

Cho tam giác $ABC$ có $H$ là trực tâm. Chứng minh: $\frac{HA}{BC}+\frac{HB}{CA}+\frac{HC}{AB}\geq \sqrt{3}$

3.JPG

 Gọi các đường cao của tam giác là $AM,BN,CP.$

 Ta có:  $\Delta APH\sim \Delta AMB(g.g)\Rightarrow \frac{AH}{AB}=\frac{AP}{AM}$

 Do đó: $\frac{HA}{BC}.\frac{HC}{AB}=\frac{HA}{AB}.\frac{HC}{BC}=\frac{AP}{AM}.\frac{HC}{BC}=\frac{\frac{1}{2}.AP.HC}{\frac{1}{2}.AM.BC}=\frac{S_{AHC}}{S_{ABC}}$

 Chứng minh hoàn toàn tương tự ta cũng được: $\frac{HA.HB}{AC.BC}=\frac{S_{AHB}}{S_{ABC}}; \;\;\;\ \frac{HB.HC}{AB.AC}=\frac{S_{HBC}}{S_{ABC}}$

 Suy ra: $\frac{HA}{BC}.\frac{HC}{AB}+\frac{HA.HB}{AC.BC}+\frac{HB.HC}{AB.AC}=\frac{S_{AHC}+S_{AHB}+S_{BHC}}{S_{ABC}}=1$

 Mà ta có BĐT quen thuộc sau: $(a+b+c)^2 \geq 3(ab+bc+ca)$. Áp dụng vào được:

$$(\frac{HA}{BC}+\frac{HB}{CA}+\frac{HC}{AB})^2\geq 3(\frac{HA}{BC}.\frac{HC}{AB}+\frac{HA.HB}{AC.BC}+\frac{HB.HC}{AB.AC})=3\Leftrightarrow \frac{HA}{BC}+\frac{HB}{CA}+\frac{HC}{AB}\geq \sqrt{3}(\text{đpcm}).$$




#650752 13 tóc đỏ, 15 tóc vàng, 17 tóc xanh. 2 có màu tóc khác nhau gặp nhau, tóc của...

Gửi bởi tpdtthltvp trong 22-08-2016 - 09:44

Ở Vương quốc "Sắc màu kì ảo'' có $45$ hiệp sĩ: $13$ hiệp sĩ tóc đỏ, $15$ hiệp sĩ tóc vàng, $17$ hiệp sĩ tóc xanh. Khi hai hiệp sĩ có màu tóc khác nhau gặp nhau, tóc của họ sẽ lập tức đổi sang màu thứ ba. Hỏi có thể có một lúc nào đó, tất cả hiệp sĩ đều có màu tóc giống nhau?

 

 Ta thấy: số các hiệp sĩ của mỗi màu tóc khác nhau khi chia cho $3$ sẽ được cả $3$ số dư $(13\equiv 1;15\equiv 0;17\equiv 2(\mod 3))$

 Nên sau mỗi lần gặp nhau, số dư của số các hiệp sĩ có màu tóc khác nhau khi chia cho $3$ vẫn có đầy đủ các số dư $0;1;2$ (Ví dụ đầu tiên hiệp sĩ tóc đỏ gặp hiệp sĩ tóc vàng thì số hiệp sĩ tóc đỏ là $12\equiv 0(\mod3)$, số hiệp sĩ tóc vàng là $14\equiv 2(\mod3)$ và số hiệp sĩ tóc xanh là $19\equiv 1( \mod3)$

 Giả sử một lúc nào đó, tất cả các hiệp sĩ có màu tóc giống nhau thì số dư khi chia cho $3$ của cả $3$ loại cùng là $0$ $(45\equiv 0\equiv 0\;\;\;\;\ (\mod3))$ (Vô lí).

 Vậy tất cả các hiệp sĩ không thể có màu tóc giống nhau ở mọi thời điểm. 

 




#650372 Tính dao động tần số của con lắc lò xo.

Gửi bởi tpdtthltvp trong 19-08-2016 - 18:06

Gọi $M,N,I$ là các điểm trên một lò xo nhẹ, được treo thẳng đứng ở trên điểm $O$ cố định. Khi lò xo có chiều dài tự nhiên thì $OM=MN=NI=10 cm.$ Găn vật nhỏ vào đầu dưới $I$ của lò xo và kích thích để vật dao động điều hòa theo phương thẳng đứng. Trong quá trình dao động, tỉ số độ lớn lực kéo lớn nhất và độ lớn lực kéo nhỏ nhất tác dụng lên $O$ bằng $3,$ lò xo dãn đều, khoảng cách lớn nhất giữa $2$ điểm $M$ và $N$ là $12 cm$ lấy $\pi ^2=10.$ Vật dao động với tần số là:

$A.2,9 Hz \;\;\;\;\;\;\;\;\;\;\ B.2,5Hz \;\;\;\;\;\;\;\;\;\;\ C.3,5 Hz \;\;\;\;\;\;\;\;\;\;\ 1,7 Hz.$




#650024 Chứng minh A là số hữu tỉ

Gửi bởi tpdtthltvp trong 17-08-2016 - 11:41

Bài 1: Biết a,b,c,d $\epsilon \mathbb{Q}$, a+b+c+d=0. Chứng minh $\sqrt{(ab-cd)(bc-da)(ca-bd)}$ là số hữu tỉ

 

Làm nốt :D

Từ giả thiết ta có $-c=a+b+d$ suy ra $-cd=ad+bd+d^2$

Do đó: $ab-cd=ab+ad+bd+d^2=(a+d)(b+d)$

Tương tự ta cũng được: $bc-da=(b+d)(c+d)$ và $ca-bd=(c+d)(a+d)$

Vậy: $\sqrt{(ab-cd)(bc-da)(ca-bd)}=\sqrt{[(a+d)(b+d)(c+d)]^2}=\left | (a+d)(b+d)(c+d) \right |$ là số hữu tỉ.




#649862 Topic: [LTDH] Mỗi ngày hai bất đẳng thức.

Gửi bởi tpdtthltvp trong 16-08-2016 - 11:56

Bài 20: Cho ba số thực dương $a,b,c$ thỏa mãn: $28(\frac{1}{a^2}+\frac{1}{b^2}+\frac{1}{c^2})=4(\frac{1}{ab}+\frac{1}{bc}+\frac{1}{ca})+2013$.

Tìm GTLN của:

$P=\frac{1}{\sqrt{5a^2+2ab+b^2}}+\frac{1}{\sqrt{5b^2+2bc+c^2}}+\frac{1}{\sqrt{5c^2+2ca+a^2}}$

Bài 20:

Từ giả thiết suy ra:

$$28(\frac{1}{a}+\frac{1}{b}+\frac{1}{c})^2=60(\frac{1}{ab}+\frac{1}{bc}+\frac{1}{ca})+2013\leq 20(\frac{1}{a}+\frac{1}{b}+\frac{1}{c})^2+2013$$

$$\Leftrightarrow 8(\frac{1}{a}+\frac{1}{b}+\frac{1}{c})^2\leq 2013\Leftrightarrow \frac{1}{a}+\frac{1}{b}+\frac{1}{c}\leq \sqrt{\frac{2013}{8}}$$

Mặt khác ta có:

$$P=\sum \frac{1}{\sqrt{5a^2+2ab+b^2}}=\sum \frac{1}{\sqrt{(2a)^2+(a+b)^2}}\leq \sum \frac{1}{\sqrt{\frac{(3a+b)^2}{2}}}=\sqrt{2}\sum \frac{1}{3a+b}$$

Do đó:

$$P\leq \sqrt{2}\sum \left [ \frac{1}{16}(\frac{3}{a}+\frac{1}{b}) \right ]=\frac{\sqrt{2}}{4}(\frac{1}{a}+\frac{1}{b}+\frac{1}{c})\leq \frac{\sqrt{2}}{4}.\sqrt{\frac{2013}{8}}=\sqrt{\frac{2013}{64}}$$

Vậy $\max P=\sqrt{\frac{2013}{64}}.$ Dấu đẳng thức xảy ra khi $a=b=c=2\sqrt{\frac{6}{671}}$




#649528 Topic: [LTDH] Mỗi ngày hai bất đẳng thức.

Gửi bởi tpdtthltvp trong 14-08-2016 - 07:43

Bài 15: Cho $a,b,c>0$. Tìm giá trị nhỏ nhất của biểu thức:$P=\frac{4a^3+3b^3+2c^3-3b^2c}{(a+b+c)^3}$

 

Áp dụng bất đẳng thức $AM-GM:$

$$2b^3+c^3=b^3+b^3+c^3\geq 3b^2c$$

Do đó:

$$P\geq \frac{4a^3+b^3+c^3}{(a+b+c)^3}$$

Theo BĐT $Holder,$ ta có:

$$(4a^3+b^3+c^3)(\frac{1}{2}+1+1)(\frac{1}{2}+1+1)\geq (a+b+c)^3\Leftrightarrow \frac{4a^3+b^3+c^3}{(a+b+c)^3}\geq \frac{4}{25}$$

Vậy $\min P=\frac{4}{25}\Leftrightarrow a=\frac{1}{2}b=\frac{1}{2}c$




#649239 Cho $a^2+b^2\vdots ab$. Tính: $A=\frac{a^2+b^2...

Gửi bởi tpdtthltvp trong 12-08-2016 - 20:05

Cho $a, b$ là các số nguyên dương thõa mãn: $a^2+b^2\vdots ab$

Tính giá trị của biểu thức: $A=\frac{a^2+b^2}{2ab}$

Giải chi tiết nhé!!

Đặt $k=\frac{a^2+b^2}{ab}$, cố định $k$ và trong các cặp $(a,b)$ nguyên dương thỏa mãn $k$ nguyên, chọn ra cặp $(a_0,b_0)$ thỏa mãn $a_0+b_0$ nhỏ nhất và $a_0 \geq b_0.$

Xét phương trình bậc $2$ ẩn $x:$    $x^2-kb_0x+b_0^2=0.$

Dễ thấy phương tình này có $1$ nghiệm là $a_0,$ gọi nghiệm còn lại là $t$ thì theo định lí $Viete,$ ta có: $\left\{\begin{matrix} t+a_0=kb_0 \;\;\ (1) \\ ta_0=b_0^2 \end{matrix}\right.$

Do tính nhỏ nhất của $a_0+b_0$ nên $t \geq a_0.$ Từ $(1)$ suy ra $kb_0\geq a_0+a_0=2a_0\Rightarrow \frac{a_0}{b_0}\leq \frac{k}{2}$

Như vậy: $k=\frac{a_0}{b_0}+\frac{b_0}{a_0}\leq \frac{k}{2}+1\Rightarrow k\leq 2$

Do đó, $k\in \left \{ 1;2 \right \}.$ mà $a^2+b^2\geq 2ab>ab$ nên $k>1.$ Từ đó $k=2$ suy ra $a=b.$ rồi tính được $A=1$

Vậy $A=1$




#648552 CMR: $\frac{AN}{NB} + \frac{AM}...

Gửi bởi tpdtthltvp trong 08-08-2016 - 12:22

Cho $\Delta ABC ; O \in \Delta ABC$; AO cắt BC tại D; BO cắt AC tại M; CO cắt AB tại N. CMR: $\frac{AN}{NB} + \frac{AM}{MC} = \frac{AO}{OD}$

 

Van Oben.PNG

Đây là định lí Van Oben mà!

Chứng minh:

Qua $A$ kẻ đường thẳng song song với $BC$ cắt $BM$ tại $P$ và $CN$ tại $Q.$

Theo định lí $Thales,$ ta có:

$$\frac{AO}{OD}=\frac{AP}{BD}=\frac{AQ}{CD}=\frac{AP+AQ}{BD+CD}=\frac{AP}{BC}+\frac{AQ}{BC}=\frac{AM}{MC}+\frac{AN}{NC}\;\;\;\;\;\ (\text{đpcm}).$$




#648248 $HF=KE$

Gửi bởi tpdtthltvp trong 06-08-2016 - 18:05

Cho tam giác nhọn $ABC$, đường cao $BE$ và $CF$. $BH$ và $CK$ vuông góc với $EF$.

Chứng minh rằng: $HF=KE$

Ta có: $\widehat{BFC}=\widehat{BEC}=90^{\circ}$ suy ra tứ giác $BFEC$ nội tiếp đường tròn $(O)$ với $O$ là trung điểm của $BC.$

Kẻ $OI$ vuông góc với $HK$ tại $I$ thì $OI$ là đường trung bình hình thang $BHKC$ nên $I$ là trung điểm của $HK. \;\;\;\;\;\;\;\ (1)$

Mà $OI$ vuông góc với dây $EF$ của $(O)$ nên $I$ là trung điểm của $EF \;\;\;\;\;\;\;\;\ (2)$

Từ $(1),(2)$ suy ra $HF=EK$ (đpcm).

 

 

NOITIEP.PNG




#648171 Chứng minh: $\sum\frac{3a+b}{2a+c}\ge...

Gửi bởi tpdtthltvp trong 06-08-2016 - 08:14

Bài 3: Cho a,b,c dương. Chứng minh:

$\frac{a-b}{a+2b+c}+\frac{b-c}{b+2c+d}+\frac{c-d}{c+2d+a}+\frac{d-a}{d+2a+b}\geq 0$

 

Bất đẳng thức cần chứng minh tương đương:

$$\sum \frac{2a-2b}{a+2b+c}\geq 0\Leftrightarrow \sum \frac{3a+c}{a+2b+c}\geq 3$$

Áp dụng $Cauchy-schwarz:$

$$\sum \frac{3a+c}{a+2b+c}=\sum \frac{(3a+c)^2}{(3a+c)(a+2b+c)}\geq \frac{16(a+b+c)^2}{\sum (3a+c)(a+2b+c)}=\frac{16(a^2+b^2+c^2)+32(ab+bc+ca)}{4(a^2+b^2+c^2)+12(ab+bc+ca)}$$

Cần chứng minh:

$$\frac{16(a^2+b^2+c^2)+32(ab+bc+ca)}{4(a^2+b^2+c^2)+12(ab+bc+ca)}\geq 3\Leftrightarrow 4(a^2+b^2+c^2)\geq 4(ab+bc+ca)\Leftrightarrow a^2+b^2+c^2\geq ab+bc+ca,\text{ đúng}$$

Suy ra đpcm. Dấu $"="$ xảy ra khi $a=b=c.$




#647672 Hỏi mỗi người đi hết quãng đường AB (.) thời gian bao lâu

Gửi bởi tpdtthltvp trong 02-08-2016 - 20:26

Tìm quan hệ giữa a,b,c để phương trình:

$(x+a)^4+(x+b)^4=c$ có nghiệm.

Từ giả thiết suy ra $c\geq 0.$

Đặt $z=x+\frac{a+b}{2}, m=\frac{a-b}{2}$ thì giả thiết viết lại thành $(z-m)^4+(z+m)^4=c\Leftrightarrow 2z^4+12m^2z^2+2m^4-c=0 \;\;\;\;\;\;\;\;\ (1)$

Lại đặt $z^2=y \;\;\ (y\geq 0)$ thì $(1)\Leftrightarrow 2y^2+12m^2y+2m^4-c=0. \;\;\;\;\ (2)$

Cần tìm mối quan hệ giữa $m,c$ để $(2)$ có nghiệm không âm.

ĐK để cả $2$ nghiệm của $(2)$ đều âm là: $\left\{\begin{matrix} \Delta '=32m^4+2c\geq 0 \\ S=x_1+x_2=-6m^2<0 \\ P=x_1x_2=\frac{2m^4-c}{2}>0 \end{matrix}\right.\Leftrightarrow  2m^4>c $ (Vì $c\geq 0$)

Vậy để phương trình có nghiệm thì: $2m^2\leq c\Leftrightarrow (a-b)^4\leq 8c$




#647586 $\sqrt{10}\left [ \left ( 1+\sqrt{10...

Gửi bởi tpdtthltvp trong 02-08-2016 - 06:44

   CM pt sau nguyên

 

$\sqrt{10}\left [ \left ( 1+\sqrt{10} \right )^{100} -\left ( 1-\sqrt{10} \right )^{100}\right ]$

 

Ps : Cho minh xin công thức chung ý.... Cách bt ra rồi

 Công thức chung: Với mọi số nguyên không âm $m,n$ và $k$ nguyên thì biểu thức sau nguyên:

$$\sqrt{m}\left [ (k+\sqrt{m})^n-(k-\sqrt{m})^n \right ]$$

 

Chứng minh:

Theo công thức khai triển $(x+y)^n$ ta có:

$(k+\sqrt{m})^n=A+B\sqrt{2}$      và     $(k-\sqrt{m})^n=A-B\sqrt{2}$ (Với $A,B\in \mathbb{N}$)

Do đó: $\sqrt{m}\left [ (k+\sqrt{m})^n-(k-\sqrt{m})^n \right ]=\sqrt{m}(A+B\sqrt{m}-A+B\sqrt{m})=\sqrt{m}.2B\sqrt{m}=2mB.$

Suy ra điều phải chứng minh.




#647563 Tìm số $\overline{a_{1}a_{2}...a_{n-1...

Gửi bởi tpdtthltvp trong 01-08-2016 - 22:11

Bài toán. Tìm số $\overline{a_{1}a_{2}...a_{n-1}a_{n}}$ thỏa mãn tính chất $\sqrt{\overline{a_{1}a_{2}...a_{n-1}a_{n}}}=a_{1}+a_{2}+...+a_{n-1}+a_{n}$.

Giả thiết tương đương: $\overline{a_{1}a_{2}...a_{n-1}a_{n}}=(a_{1}+a_{2}+...+a_{n-1}+a_n)^2$

Suy ra: $\overline{a_{1}a_{2}...a_{n-1}a_{n}}\leq (9n)^2=81n^2.$ Ta sẽ chứng minh với $n\geq 5$ thì $\overline{a_{1}a_{2}...a_{n-1}a_{n}}> (9n)^2\;\;\;\;\;\;\;\(1)$

Thật vậy, với $n=5$ thì $(1)$ đúng. Giả sử đúng với $n=k(k\geq 5),$ thì $\overline{a_{1}a_{2}...a_{n-1}a_{k}}> 81k^2.$Ta đi chứng minh đúng với $n=k+1.$ Với $n=k+1,$ ta có: $(1)\Leftrightarrow \overline{a_{1}a_{2}...a_{n-1}a_{k+1}}> 81(k+1)^2\Leftrightarrow 10.\overline{a_{1}a_{2}...a_{n-1}a_{n}}+a_{k+1}>81k^2+162k+81$

Mà $\overline{a_{1}a_{2}...a_{n-1}a_{n}}>81k^2\Rightarrow 10.\overline{a_{1}a_{2}...a_{n-1}a_{n}}+a_{k+1}>810k^2.$ Ta thấy $810k^2>81k^2+162k+81\Leftrightarrow 719k^2-162k-81>0.$ Đặt $f(x)=719k^2-162k-81$ thì $f(x)$ đồng biến trên $\left [ 5;\infty+  \right ]$ nên $f(x)\geq f(5)>0.$

Từ đó suy ra $n\leq 4\Rightarrow n\in \left \{ 1;2;3;4 \right \}.$

  • $n=1:$ thì $a_1=a_1^2\Rightarrow a_1=1.$ Số cần tìm là $1$
  • $n=2:$ thì $\overline{a_1a_2}=(a_1+a_2)^2.$ Để ý $a_2\in \left \{ 0;1;4;5;6;9 \right \}.$ Thế vào phương trình $10a_1+a_2=(a_1+a_2)^2$ được số cần tìm là $81$
  • $n=3:$ thì $\overline{a_1a_2a_3}=(a_1+a_2+a_3)^2\Rightarrow 10\leq a_1+a_2+a_3\leq 27.$ Thử từng TH thấy không có số nào thỏa mãn :D
    P/S
  • $n=4$ thì $\overline{a_1a_2a_3a_4}=(a_1+a_2+a_3+a_4)^2\Rightarrow 31\leq a_1+a_2+a_3+a_4\leq 36.$ Cái này ít TH nên thử được, cũng vô nghiệm.

Vậy các số cần tìm là $1;81$